Overunity.com Archives is Temporarily on Read Mode Only!



Free Energy will change the World - Free Energy will stop Climate Change - Free Energy will give us hope
and we will not surrender until free energy will be enabled all over the world, to power planes, cars, ships and trains.
Free energy will help the poor to become independent of needing expensive fuels.
So all in all Free energy will bring far more peace to the world than any other invention has already brought to the world.
Those beautiful words were written by Stefan Hartmann/Owner/Admin at overunity.com
Unfortunately now, Stefan Hartmann is very ill and He needs our help
Stefan wanted that I have all these massive data to get it back online
even being as ill as Stefan is, he transferred all databases and folders
that without his help, this Forum Archives would have never been published here
so, please, as the Webmaster and Creator of this Forum, I am asking that you help him
by making a donation on the Paypal Button above
Thanks to ALL for your help!!


The Lorentz Force is an APPROXIMATION

Started by kmarinas86, January 13, 2014, 03:41:04 AM

Previous topic - Next topic

0 Members and 1 Guest are viewing this topic.

kmarinas86

This is key to understanding various energetic anomalies involving unusual
electromagnetic devices, including Joseph Newman's energy machine and Bruce DePalma's N-
machine.
https://web.archive.org/web/20140113061513/http://www.scielo.br/scielo.php?pid=S1806-11172012000200006&script=sci_arttext

// "
4. Conclusions
We have shown in the present work that, by using the vectorial identities, Eqs. (9)-(11),
it is possible to obtain directly from Maxwell's equations the momentum balance equation,
identifying in it a force density that we have called Maxwell's force density. Robinson's
criticism [9] that this type of deduction is a mere identity does not constitute a
demerit; otherwise we would have to blame also the continuity equation of charge and
current, since it also results from an identity derived from Maxwell's equations.
It is also clear from this deduction that the usual Lorentz force is an approximation for
very small charges and corresponding currents, which permits to neglect the self fields.
Therefore the generalized Lorentz force, including total fields, is implicit in
Maxwell's equations.
The inclusion of total fields is essential to the study of
radiation reaction. On the other hand such a general result like this must be tested with
particular models and experiments. In this sense is remarkable that the Helmholtz force
density for fluids is contained, as a particular case, in the Maxwell balance equation.
This is a relevant result not appearing in the literature. A final point, relevant for
teaching, is the use of vector identities, not all familiar to students that permit a
straightforward deduction, avoiding in this way the tricky deductions with components
that obscure the deduction and make it repellent to the reader.
" //

The significance of this:
1) The current of a conductor does not act directly on an external magnetic field.
2) The magnetic field of current of a conductor does act directly on an external
magnetic field.
The Lorentz force calculates the force on a wire based on the field strength through that
wire. However, the magnetic field of the current of the wire extends far beyond the
limits of the wire. How this matters is based on the fact that the magnetic field of a
magnet is a function of position. There are different pairs of points on the magnetic
field of a magnet where the magnetic field strength points are mutually opposite in
direction. Since the magnetic field of a current extends to infinity and the magnetic
field of a magnet extends to infinity, then their forces are distributed throughout all
of space. Only a calculation of the forces over all of space, as the spatial integral of
force density, using Maxwell's equations, should give the correct answer. The Lorentz
force is thus not only redundant and can be accounted for by applying a space integral,
it is also inaccurate.

The only reason why the Lorentz force should be used is when it doesn't make a
difference. The reasons why it is used is probably due to the ease of calculating
a single path integral vs. the difficulty of calculating a four-integral that combines a
path integral with a space integral.

In the case of the Newman machine:
There is a range of angles close to top dead center (TDC) where the induced magnetic
field due to rotation of the magnet field of the magnet through the conducting coil is
actually reverse of what would slow down the magnet. To see why, visualize the magnetic
field around a magnet. Then imagine conductors passing through this field.

https://sites.google.com/site/kmarinas86/energy/newman-machine/Newman61.png

As might be expected in any motor, some of the turns of the coil will have induced in
them voltages opposed to that of other turns in the same coil. If we position the magnet
so that more of the lines that cut through the conductor induce current in the
other direction, the induced magnetic field of the coil will actually repel the
magnet away from top dead center if the magnet is rotating from top dead center toward
the coil, while in the other case, where the magnet moves toward top dead center when it
is already close to it, the induced magnetic field will actually pull the magnet toward top
dead center. The problem with this is that this effect only occurs in a small angle, so
it becomes important to reuse this energy for other parts of the rotation. In addition,
this effect turns out to be very weak, as the peripheral field is less energy dense. So
over the course of a full rotation, the back-emf tends to dwarf the forward emf.

Below is an image of the Newman machine slightly off to top dead center:

https://sites.google.com/site/kmarinas86/energy/newman-machine/Newman70.png

As I stated, the magnetic field of the magnet does not act directly on a current. It acts
on the magnetic field of that current, and because it may vary in direction in intensity
and direction throughout space, the Lorentz force is false and at best an
approximation. However, a changing magnetic field produces an electric field. It is the
change of this magnetic field (related to the time derivative of the magnetic vector
potential) which produces the induced current. A function is not generally the same as
its time derivative.
The forces that drive the relative motion between the magnet and
the wire are technically not the same as those forces that drive the current. The
magnetic vector potential is technically what drives the induced emf. Look at the fact of
momentum conservation. For every force is an equal and opposite force. So knowing that
the force between the magnet and the conductor is essentially at a right angle to the
acceleration of charge due to induced emf, this means the forces between the magnet and
the wire do not account for what happens in the wire itself. Why? Total momentum
conservation requires that all forces must balance. If your problem does not conserve
momentum, then either you did the problem wrong, or there is something else that
exists which balances this equation. In this case it is the magnetic vector potential of
a line of magnetic flux; this does line up on the same axis as the induced current. The
force against (or in this case, with) the rotating magnet, a force which is tangential to
the shaft rotation, cannot account for the force which is parallel to the current, as they
are essentially perpendicular. So the force that drives the current exists on what is
essentially a different force channel than what drives the force between the
magnetic field of the magnet and the magnetic field of the current, a force that acts
perpendicularly to both the magnetic field and the current which produces it.
Below is an image of the Newman machine version with the magnet outside:

https://sites.google.com/site/kmarinas86/energy/newman-machine/Newman43.png

As far as the N-machine is concerned, you must realize that the Lorentz force is
essentially false
(and so is the Laplace force). The key is in finding that the
magnetic field that is parallel with the shaft is not being rearranged as it spins due to
the symmetry of the magnet, so essentially there is no force between it and the magnetic
field of the wire that would cause it to speed up or slow down. The magnetic vector
potential, which is reflected off the electrons and thereby produces a current acts on a
completely separate axis and cannot slow down the rotation of the magnet.

MarkE

Kmarinas, Joseph Newman's energy machine was subjected to very detailed tests by the NBS, now NIST, who found that it did not generate surplus energy.  No one has shown surplus energy from Bruce DePalma's machine either.  The Lorentz Force has been subjected to very careful evaluation by NBS, and Ampere balances were used for many years to calibrate weights.  If the Lorentz force varied even by a small amount, those weight calibrations would be off.

If you wish to argue that the Lorentz Force is inaccurate, then you should propose and execute an experiment that would show that.

Marsing



approximation!!,
if everyone accept this...   

PiCéd

If the Lorentz for is conservative then somebody must certainly have a new equation to prove that.
I realy think that this force is conservative but to do that his equation must be 0.

kmarinas86

You might be interested in the following pictures. These are also posted at http://www.energeticforum.com/renewable-energy/16504-snap-wrap-bracelet-visualization-magnetic-induction.html.

To be more precise about the third image (http://www.overunity.com/14200/the-lorentz-force-is-an-approximation/dlattach/attach/131982/image//), the right-hand example does not violate the Lorentz force per se, as the rules for electromagnetic induction used in the example imply the Lorentz force (which is exerted in the direction of charge acceleration due to relative motion of the magnetic field through the conductor). But what it does violate is Fleming's right-hand rule for generators. The force between the magnet and the current it induces is the same you would expect if the system were being driven as a motor. This is possible due to the special placement of the magnet and the windings (very untypical).